Last visit was: 25 Apr 2024, 22:02 It is currently 25 Apr 2024, 22:02

Close
GMAT Club Daily Prep
Thank you for using the timer - this advanced tool can estimate your performance and suggest more practice questions. We have subscribed you to Daily Prep Questions via email.

Customized
for You

we will pick new questions that match your level based on your Timer History

Track
Your Progress

every week, we’ll send you an estimated GMAT score based on your performance

Practice
Pays

we will pick new questions that match your level based on your Timer History
Not interested in getting valuable practice questions and articles delivered to your email? No problem, unsubscribe here.
Close
Request Expert Reply
Confirm Cancel
SORT BY:
Kudos
Tags:
Show Tags
Hide Tags
SVP
SVP
Joined: 14 Apr 2009
Posts: 2261
Own Kudos [?]: 3671 [5]
Given Kudos: 8
Location: New York, NY
Send PM
User avatar
Manager
Manager
Joined: 10 Apr 2013
Posts: 98
Own Kudos [?]: 66 [1]
Given Kudos: 17
Send PM
User avatar
Manager
Manager
Joined: 21 Jan 2010
Posts: 193
Own Kudos [?]: 610 [0]
Given Kudos: 12
Send PM
User avatar
Manager
Manager
Joined: 21 Aug 2012
Posts: 82
Own Kudos [?]: 168 [0]
Given Kudos: 41
Send PM
Re: Higher rates of respiratory problems in Clark have been linked to the [#permalink]
Bluelagoon wrote:
GMATPill wrote:
Which of the following most logically completes the passage?

Higher rates of respiratory problems in Clark have been linked to the airborne pollutants being released from the Lauriel cosmetic manufacturing plant. In response to these new findings, the city imposed regulations on the Lauriel plant, requiring it to reduce emissions by half in two years’ time. While Lauriel has no problem meeting these new emission levels, it is unlikely that the rate of respiratory problems two years from now will be reduced since ______.

(A) the number of facilities capable of treating respiratory ailments is not likely to increase
(B) reducing emissions even further than suggested through regulation would necessitate decreasing production at Lauriel
(C) it is difficult to make accurate, long-term predictions about emissions
(D) not all respiratory ailments are caused by airborne pollutants
(E) two new cosmetic manufacturing plants are about to go into production in Clark


The answer is E as it clearly identifies why imposing limits on Lauriel plant will not work. The limit should be applied in general. here its specific.


Why E... There is no guarantee that these two new cosmetic manufacturing plants will produce pollution... They might use some different process...
IMO D..
avatar
Intern
Intern
Joined: 30 Oct 2011
Posts: 25
Own Kudos [?]: 64 [0]
Given Kudos: 13
Send PM
Re: Higher rates of respiratory problems in Clark have been linked to the [#permalink]
jaituteja wrote:
Bluelagoon wrote:
GMATPill wrote:
Which of the following most logically completes the passage?

Higher rates of respiratory problems in Clark have been linked to the airborne pollutants being released from the Lauriel cosmetic manufacturing plant. In response to these new findings, the city imposed regulations on the Lauriel plant, requiring it to reduce emissions by half in two years’ time. While Lauriel has no problem meeting these new emission levels, it is unlikely that the rate of respiratory problems two years from now will be reduced since ______.

(A) the number of facilities capable of treating respiratory ailments is not likely to increase
(B) reducing emissions even further than suggested through regulation would necessitate decreasing production at Lauriel
(C) it is difficult to make accurate, long-term predictions about emissions
(D) not all respiratory ailments are caused by airborne pollutants
(E) two new cosmetic manufacturing plants are about to go into production in Clark


The answer is E as it clearly identifies why imposing limits on Lauriel plant will not work. The limit should be applied in general. here its specific.


Why E... There is no guarantee that these two new cosmetic manufacturing plants will produce pollution... They might use some different process...
IMO D..


Though E is OA, I don't get why D is not correct. Anyone please help with the explanation. Thanks!
User avatar
Manager
Manager
Joined: 25 Sep 2012
Posts: 204
Own Kudos [?]: 557 [0]
Given Kudos: 242
Location: India
Concentration: Strategy, Marketing
GMAT 1: 660 Q49 V31
GMAT 2: 680 Q48 V34
Send PM
Re: Higher rates of respiratory problems in Clark have been linked to the [#permalink]
About D, yes, It is a good choice but E is Better.

If you go with D, you are trying to question the impact of the airborne pollutants being released from the Lauriel cosmetic manufacturing plant.
You are trying to deny the first sentence as it clearly says that airborne pollutants are linked with respiratory problems.
SVP
SVP
Joined: 14 Apr 2009
Posts: 2261
Own Kudos [?]: 3671 [0]
Given Kudos: 8
Location: New York, NY
Send PM
Re: Higher rates of respiratory problems in Clark have been linked to the [#permalink]
Yes, that's a good point.

In terms of frameworks, here's how we think about it at GMAT PILL:

A leads to B

[Lauriel plant emissions] leads to [respiratory problems]

The passages suggests that emissions will go down.

So given that (A --> B), if we reduce/remove A - then one might think that B disappears.

If we reduce emissions, one might think that respiratory problems will disappear.

But the author says that will UNLIKELY happen because.... why?

Because something else OTHER than A (let's call it C) will lead to B.

If we have something else that leads to respiratory problems, then removing Lauriel emissions will not necessarily solve the respiratory problem.

That's exactly what (E) says.

(E) says that 2 NEW plants will come on - and that will lead to respiratory problems.

So even if we remove the original Lauriel emissions, those 2 new plants will still lead to respiratory problems.

So this is an example of the Linked Chains Frameworks between A, B, and C.

A leads to B.

Negate it and you might expect that:

(Without A) --> there will be no B.

But actually, if something else C leads to B, then removing A will not necessarily lead to no B.

Because C will still lead to B.

Does that make sense?

You can learn more about Linked Chains in the Critical Reasoning Pill Frameworks.

User avatar
Intern
Intern
Joined: 05 Jun 2013
Posts: 35
Own Kudos [?]: 16 [0]
Given Kudos: 31
Location: India
Concentration: Entrepreneurship, General Management
GMAT 1: 640 Q48 V29
GPA: 3.6
WE:Business Development (Education)
Send PM
Re: Higher rates of respiratory problems in Clark have been linked to the [#permalink]
@GMATPill

Can we not consider
Lauriel cosmetic --> airborne pollutants -->Higher rates of respiratory problems

A --> B --> C

But looking at option D; [not all respiratory ailments are caused by airborne pollutants]

C can be caused by D.
D --> C

Please explain. (Options are too close. )
avatar
Intern
Intern
Joined: 14 Feb 2012
Posts: 16
Own Kudos [?]: 17 [0]
Given Kudos: 6
Send PM
Re: Higher rates of respiratory problems in Clark have been linked to the [#permalink]
GMATPill wrote:
Yes, that's a good point.

In terms of frameworks, here's how we think about it at GMAT PILL:

A leads to B

[Lauriel plant emissions] leads to [respiratory problems]

The passages suggests that emissions will go down.

So given that (A --> B), if we reduce/remove A - then one might think that B disappears.

If we reduce emissions, one might think that respiratory problems will disappear.

But the author says that will UNLIKELY happen because.... why?

Because something else OTHER than A (let's call it C) will lead to B.


If we have something else that leads to respiratory problems, then removing Lauriel emissions will not necessarily solve the respiratory problem.

That's exactly what (E) says.

(E) says that 2 NEW plants will come on - and that will lead to respiratory problems.

So even if we remove the original Lauriel emissions, those 2 new plants will still lead to respiratory problems.

So this is an example of the Linked Chains Frameworks between A, B, and C.

A leads to B.

Negate it and you might expect that:

(Without A) --> there will be no B.

But actually, if something else C leads to B, then removing A will not necessarily lead to no B.

Because C will still lead to B.

Does that make sense?

You can learn more about Linked Chains in the Critical Reasoning Pill Frameworks.



Can you not even see for yourself that something else that leads to b is perfectly addressed in option D?
I dont get the point why you are developing such amibigous questions at the CR part.

Most of the CR questions you develop are just slightly changed questions from the OG.
While you are doing decent things at other domains, your CR part is just awful.

You are not doing anyone a favour with posting these types of CR questions, as CR questions at test day will be very subtle, and answer choices above all will be clearly structured.
User avatar
Manager
Manager
Joined: 07 Apr 2012
Posts: 71
Own Kudos [?]: 25 [0]
Given Kudos: 45
Location: United States
Concentration: Entrepreneurship, Operations
Schools: ISB '15
GMAT 1: 590 Q48 V23
GPA: 3.9
WE:Operations (Manufacturing)
Send PM
Re: Higher rates of respiratory problems in Clark have been linked to the [#permalink]
For me its D. There is no certainty that new plants will also add up the pollution.
But having an alternate reason for existing pollution levels would mean shutting down plant may not have intended outcome.
Intern
Intern
Joined: 31 Oct 2011
Posts: 45
Own Kudos [?]: 97 [0]
Given Kudos: 25
Location: India
GMAT 1: 660 Q48 V32
GPA: 3.56
WE:Programming (Computer Software)
Send PM
Re: Higher rates of respiratory problems in Clark have been linked to the [#permalink]
waltiebikkiebal wrote:
GMATPill wrote:

If we have something else that leads to respiratory problems, then removing Lauriel emissions will not necessarily solve the respiratory problem.

That's exactly what (E) says.

(E) says that 2 NEW plants will come on - and that will lead to respiratory problems.



E does not explicitly state that the two new cosmetic manufacturing plants lead to respiratory problems by releasing airborne pollutants, not does the passage state so. According to the passage,

"Higher rates of respiratory problems in Clark have been linked to the airborne pollutants being released from the Lauriel cosmetic manufacturing plant"

The respiratory problems are linked to pollutants released fro Lauriel cosmetic manufacturing plant only. Are we not generalizing this statement to all cosmetic manufacturing plants to arrive at our answer? It could be true that it is only Lauriel cosmetic manufacturing plant releases airborne pollutants and all other plants already follow strict norms and are likely to incorporate the same in their two new plants as well.
Intern
Intern
Joined: 31 Oct 2011
Posts: 45
Own Kudos [?]: 97 [0]
Given Kudos: 25
Location: India
GMAT 1: 660 Q48 V32
GPA: 3.56
WE:Programming (Computer Software)
Send PM
Re: Higher rates of respiratory problems in Clark have been linked to the [#permalink]
ygdrasil24 wrote:
For me its D. There is no certainty that new plants will also add up the pollution.
But having an alternate reason for existing pollution levels would mean shutting down plant may not have intended outcome.


Similarly, even D appears debatable.

It says "not all" respiratory ailments are caused by airborne pollutants. Even if this were true, not all just means, not 100%. It could be anything from 0% to 99.9%. Let's say 60%. If 60% of respiratory ailments are caused by airborne pollutants which are in turn released from the Lauriel cosmetic manufacturing plant, it is no more an explanation why the rate of respiratory problems will not reduce over the next two years if the emissions are reduced.
VP
VP
Joined: 18 Dec 2017
Posts: 1170
Own Kudos [?]: 991 [0]
Given Kudos: 421
Location: United States (KS)
GMAT 1: 600 Q46 V27
Send PM
Re: Higher rates of respiratory problems in Clark have been linked to the [#permalink]
GMATPill wrote:
Which of the following most logically completes the passage?

Higher rates of respiratory problems in Clark have been linked to the airborne pollutants being released from the Lauriel cosmetic manufacturing plant. In response to these new findings, the city imposed regulations on the Lauriel plant, requiring it to reduce emissions by half in two years’ time. While Lauriel has no problem meeting these new emission levels, it is unlikely that the rate of respiratory problems two years from now will be reduced since ______.

(A) the number of facilities capable of treating respiratory ailments is not likely to increase
(B) reducing emissions even further than suggested through regulation would necessitate decreasing production at Lauriel
(C) it is difficult to make accurate, long-term predictions about emissions
(D) not all respiratory ailments are caused by airborne pollutants
(E) two new cosmetic manufacturing plants are about to go into production in Clark

original source:Practice Pill Platform


GMATNinja, I don't want to bother you for this old question but if you can quickly help pick between D and E, that would be greatly appreciated. Thank you!
GMAT Club Bot
Re: Higher rates of respiratory problems in Clark have been linked to the [#permalink]
Moderators:
GMAT Club Verbal Expert
6921 posts
GMAT Club Verbal Expert
238 posts
CR Forum Moderator
832 posts

Powered by phpBB © phpBB Group | Emoji artwork provided by EmojiOne